Add illustration for Serway and Jewett v8's problem 23.09.
authorW. Trevor King <wking@drexel.edu>
Tue, 3 Apr 2012 14:12:29 +0000 (10:12 -0400)
committerW. Trevor King <wking@drexel.edu>
Tue, 3 Apr 2012 14:12:29 +0000 (10:12 -0400)
latex/problems/Serway_and_Jewett_8/problem23.09.tex

index 66f6da162aca49f0b8d98a450bee4a530ab5f2cd..8113e347ce346849c392d21d1c73be1a347796f4 100644 (file)
@@ -2,10 +2,27 @@
 Three point charges are arranged as shown in Figure~P23.9.
 Find \Part{a} the magnitude and \Part{b} the direction of the electric
 force on the particle at the origin.
+\begin{center}
 % y5.00nC      6.00nC
 % o-----------o----x
 % |0.100m  0.300m
 % o-3.00nC
+\begin{asy}
+import Mechanics;
+import ElectroMag;
+
+real u = 16cm;
+
+Charge a = pCharge((0,0), 5, L="$5.00\U{nC}$");
+Charge b = pCharge((0.3u,0), 6, L="$6.00\U{nC}$");
+Charge c = nCharge((0,-0.1u), -3, L="$-3.00\U{nC}$");
+Distance dab = Distance(a.center, b.center, scale=u, L="$0.300\U{m}$");
+Distance dac = Distance(
+    a.center, c.center, offset=-0.1u, scale=u, L="$0.100\U{m}$");
+a.draw(); b.draw(); c.draw(); dab.draw(); dac.draw();
+draw_ijhat((-0.1u,-0.08u));
+\end{asy}
+\end{center}
 \end{problem*}
 
 \begin{solution}